LSAT and Law School Admissions Forum

Get expert LSAT preparation and law school admissions advice from PowerScore Test Preparation.

 Administrator
PowerScore Staff
  • PowerScore Staff
  • Posts: 8916
  • Joined: Feb 02, 2011
|
#40557
Complete Question Explanation
(The complete setup for this game can be found here: lsat/viewtopic.php?t=15423)

The correct answer choice is (A)

This question stem requires you to replace the second rule, which created the following relationship:
PT70 -Game_#1_#7_diagram 1.png

The rule itself is fairly straightforward, and operationally limits X from performing in the first few slots and W and Z from performing in the last slot. Because the wording of the correct answer can be difficult-to-impossible to predict, after quickly examining the effects of the rule, move on to the answer choices and consider each in turn. But keep in mind that the second rule is linked to the first rule via Z, and thus that the effects of the second rule extend beyond the immediate Not Laws created by the sequence.

Answer choice (A): This is the correct answer choice. At first glance, this may not seem like a contender, but then again there is no immediate reason to discount this answer choice, and thus most students end up keeping this as a Contender and then returning to it after eliminating each of the other answer choices.

As mentioned during the setup, X is restricted to performing in either slot five or slot six. This occurs because X is forced to perform later than W, Z, V (because Z is forced to perform later than V), and Y (from the fourth rule). If you eliminate the second rule and replace it with the condition in this answer choice, X is again forced to appear later than W, Z, V, and Y. Thus, the effect is identical, and this is the correct answer choice.

Answer choice (B): This answer choice is incorrect because the rule it creates—V :longline: W :longline: Z—does not limit X in any manner, and because a new restriction is created between W and Z that did not previously exist.

Answer choice (C): This answer choice simply substitutes V for Z in the second rule.
PT70 -Game_#1_#7_diagram 2.png

With the first rule still in effect (V :longline: Z), this is closer to being correct than the prior answer, but now there is no relationship between Z and X, which would allow both Z and X greater range of movement than they had under the original rule. Thus, this answer choice is incorrect.

Answer choice (D): This answer choice creates a rotating XU not-block. This creates a new limitation in the game, and also does not emulate the limitation created by the original rule. Thus, this answer choice is incorrect.

Answer choice (E): This can initially be an attractive answer choice. While it matches the restriction on the placement of X, it does not limit only U to performing after X. For example Z or W could perform later than X under this rule, which is impossible in the original rule that is being replaced. Thus, this answer choice is incorrect.
You do not have the required permissions to view the files attached to this post.
 reop6780
  • Posts: 265
  • Joined: Jul 27, 2013
|
#16849
The correct answer is A while I chose E.

This was especially tricky since both A and E were right based upon the set up that I made.

The problem is to figure out which one reflects the rule that both W and Z perform earlier than X.

I thought this rule created Not Laws for X so that X cannot be in 1-3 in combination with the rule that V is earlier than Z.

With combination with another rule that U should be in the last three spots, X is left to fill either 5 or 6.

At the same time, Not Laws create dual option for 6 that only U and X can fill in.

Is answer A more comprehensive than answer E?

Is answer E short of a content that imply that only U or X should fill in 6?

In this kind of problem, should the focus be whether the answer reflects most of the characteristics induced by the rule?
 Robert Carroll
PowerScore Staff
  • PowerScore Staff
  • Posts: 1787
  • Joined: Dec 06, 2013
|
#16902
reop,

For a Rule Substitution question like this, you need a rule that exhibits all and only the restriction of the original rule. Answer choice (E) gets you that X is in one of the final two slots, but what if X is 5 and Z 6? That's consistent with answer choice (E) but not the original rules. Answer choice (A) makes X 5 or 6, and if it's 5, only U could be 6. Thus, neither W nor Z could possibly be 6th, after X.

Robert Carroll
 elysia
  • Posts: 6
  • Joined: Sep 18, 2016
|
#31111
Dear Powerscore Admin,

I tend to run into trouble when it comes to Rule Substitution questions. For this question, I noticed that X was restricted to slots 4, 5, or 6 which rules out answers D, E, but from then on I wasn't sure how to approach answers A. B, and C.

In general, what is the best way to attack a rule substitution question?

Thank you!
 Adam Tyson
PowerScore Staff
  • PowerScore Staff
  • Posts: 5153
  • Joined: Apr 14, 2011
|
#31121
Rule substitution questions like this one are all about rebuilding the original diagram. Start by removing the original rule, and any inferences that came out of that rule. Then, look for a rule that ends up putting the diagram back exactly the same way. One approach is to do a new diagram from scratch, without the removed rule, and then consider how you would diagram each answer choice. If you would diagram it and its related inferences any way other than what you did with the original rule, it's the wrong answer. When you find an answer that puts everything back the way it was, you have a winner!

Give that a try with this question - rebuild your original diagram, minus the removed rule, and then see which answer choice gets you back to your point of origin.

Good luck!

Get the most out of your LSAT Prep Plus subscription.

Analyze and track your performance with our Testing and Analytics Package.